Multivariable Calc Image-Graph Problem

  • Thread starter Thread starter Loppyfoot
  • Start date Start date
  • Tags Tags
    Multivariable
Click For Summary
The discussion revolves around converting a parametric curve defined by the function f(t) = (t, t^2 - cos(t)) into a graph representation in R3. Participants clarify that the graph can be expressed as F(t) = (t, t, t^2 - cos(t)). For the second part, they discuss finding a function h such that its level set for k=0 corresponds to the same curve, leading to the equation (t, t^2 - cos(t)) = 0. There is confusion about how to derive an implicit equation from the level set, with participants suggesting that the first step involves solving for t. The conversation highlights the challenges of visualizing and transforming parametric equations into different mathematical forms.
Loppyfoot
Messages
192
Reaction score
0
Multivariable Calc Image-Graph Problem!

Homework Statement



Given a function f: R-R2 , by f(t) = (t, t^2 - cos(t)), which represents a curve in the xy plane parametrically, give a function whose GRAPH represents this same curve.

2) Also, give a function h whose level set for height k=0 represents this same curve.


The Attempt at a Solution



From what I know, the graph of this function would be in R3. How would I make a function of a graph from that function? I'm probably missing an easy point but haven't picked up on it yet.

For number 2, I'm pretty confused on where to begin this problem.

Any thoughts?

Thanks a ton!
 
Physics news on Phys.org


Yes, the graph of F:t-> (x(t), y(t)) is (t, x(t), y(t)). Just put in the x(t) and y(t) you are given.

For 2 you want a function z= F(x, y) such that F(t, t^2- cos(t))= 0 for all t.
 


Ok Thank you HallsofIvy.

So for another example for number one would be:

If I have f(u,v) from R2-R3, = (u^2+v,u+v,u+v^2),

The graph of this function would be: (u,v, U^2+v,u+v, u+v^2).

Great, I understand now! Thanks a lot sir.
 


So the equation would be f(t)= (t,t,t^2-cos(t)), correct? Just making sure...
 


I'm having trouble with number 2.

So I understand that for level sets, I look at the outputs and set them equal to some k.

In my case, k=0.

So, (t,t^2-cos(t))= 0.

Should the first step be to solve for t?

If so, how would I create an implicit equation from this problem? Thanks!
 
Question: A clock's minute hand has length 4 and its hour hand has length 3. What is the distance between the tips at the moment when it is increasing most rapidly?(Putnam Exam Question) Answer: Making assumption that both the hands moves at constant angular velocities, the answer is ## \sqrt{7} .## But don't you think this assumption is somewhat doubtful and wrong?

Similar threads

  • · Replies 19 ·
Replies
19
Views
2K
  • · Replies 3 ·
Replies
3
Views
3K
  • · Replies 11 ·
Replies
11
Views
3K
  • · Replies 4 ·
Replies
4
Views
1K
  • · Replies 19 ·
Replies
19
Views
1K
  • · Replies 3 ·
Replies
3
Views
2K
  • · Replies 2 ·
Replies
2
Views
2K
  • · Replies 8 ·
Replies
8
Views
2K
  • · Replies 1 ·
Replies
1
Views
9K
  • · Replies 1 ·
Replies
1
Views
1K